Write an equation that you can use to solve for x. Enter your answer in the box. ​

Answers

Answer 1
The answer is X=27 degrees

Steps-
90-63=27
Answer 2

Answer:

x+63 = 90

Step-by-step explanation:

x and 63 add to a right angle so their sum is 90

x+63 = 90

We can solve

x = 90-63

x = 27


Related Questions

The ages, in years, of employees at a fabric store are represented by the box-and-whisker plot shown.

Answers

Answer:

There isn't a plot.

Step-by-step explanation:

PLEASE HELP ME WHAT IS THE ANWSER

Answers

Answer:

c is the same angle as a

b and d are the same as the other

and together the are what's left from 360° after a and c got their share

so

360 -98 -98 = b + d

makes

164 = b + d

since they are of equal value, just decide 164 by 2 and you got b and d at once

pls brainliest

Answer:

Vertical Angles: a and c , d and b

m<b= 82⁰ , m<c=98⁰ , m<d=82⁰

Adjacent Angles: a and b , b and c , c and d ,d and a

correct me if im wrong

Can someone help me with this math problem ?? Pls it’s for my final !
-
A 525ft cable runs from the top of an antena to the ground. The angle of elevation made by the ground to the top of an antena 25°
How tall is the antena ??

Answers

Answer:

221.87 feet

Step-by-step explanation:

Given that,

A 525 ft cable runs from the top of an antenna to the ground.

The angle of elevation made by the ground to the top of an antena 25°

We need to find the height of the antenna.

Using trigonometry,

Hypotenuse, H = 525 ft

θ = 25°

So,

[tex]\sin\theta=\dfrac{P}{H}\\\\P=H\sin\theta\\\\P=525\times \sin(25)\\\\P=221.87\ ft[/tex]

So, the height of the antenna is equal to 221.87 feet.

Help me please!!! I really need it!

Answers

Answer: The value of x can be either 5 or 10.

Step-by-step explanation:

We are given:

Total profit, P = $5600

The given equation follows:

⇒ P = 600 + 1500x - 100x²

Putting value of P in above equation, we get:

⇒ -100x² + 1500x + 600 = 5600

⇒ -100x² + 1500x - 5000 = 0

⇒ 100x² - 1500x + 5000 = 0

Solving this equation by middle term split, we get:

⇒ 100(x² - 15x + 50) = 0

⇒ (x² - 10x - 5x + 50) = 0

⇒ x(x - 10) - 5(x -10) = 0

⇒ (x - 10)(x - 5) = 0

⇒ x = 10, 5

Hence, the value of x can be either 5 or 10.

Martha’s biweekly gross pay is $1,000. Her deductions make up 23% of her gross pay. How much is deducted from her pay for a full calendar year?
$4,600
$5,980
$7,020
$9,420
ALL GIVE BRAINLY

Answers

Answer:

$5,980

Step-by-step explanation:

23% of $1,000 = $230

so, Martha's deductions for a full calendar year =

52/2 × $230 = $5,980

write an equation then find the angle measure for A. 25 points and brainliest.

Answers

Answer:

A

Step-by-step explanation:

The sum of the 3 angles in a triangle = 180°

Sum the 3 angles and equate to 180

4 + 3x + 3x + 14 + 90 = 180 , simplify left side

108 + 6x = 180 ( subtract 108 from both sides )

6x = 72 ( divide both sides by 6 )

x = 12

Then

∠ A = 4 + 3x = 4 + 3(12) = 4 + 36 = 40° → A

please help asap! i will mark brainliest!!!!!!

Answers

Answer:

it is A

Step-by-step explanation:

A is the answer, your welcome!

Which polynomial is factored completely?

Answers

Answer: fourth answer

Step-by-step explanation:

you cant factor it

Pls help and give me right anwser

Answers

Answer:

Step-by-step explanation:

s+1+2s-1+90=180 degree(being linear pair)

3s+90=180

3s=180-90

s=90/3

s=30 degree

Answer:

s = 30

Step-by-step explanation:

the total angle is a straight angle that's composed of a right angle and 2 unknown angles.

straight angle = 180°right angle = 90°

(s + 1) + 90 + (2s- 1) = 180

s + 2s + 90 + 1 - 1 = 180

3s = 180 - 90

3s = 90

s = 90/ 3

s = 30

Pre-calc, image attached

Answers

Answer:

[tex]\lim_{x \to \(-1} f(x)=-2[/tex]

Step-by-step explanation:

In order for a limit to exist in a piecewise function, both its right-side and left-side limits must be equal to each other:

So, if we plug in x=-1 for the left side limit, we get 2cos(πx) = 2cos(π(-1)) = 2cos(-π) = 2(-1) = -2Plugging in x=-1 for the right-side limit, we get 2/cos(πx) = 2/cos(π(-1)) = 2/cos(-π) = 2/(-1) = -2Since both the right-side and left-side limits are equal to each other, we can say that the limit of f(x) as x approaches -1 is -2, therefore the limit existsSee the attached graph

Can you explain how you answered it thank you!
x=
y=

Answers

Answer:

x = 1.5 or 1 1/2

y = 8

Step-by-step explanation:

Solving for x:

2x + 1/2 y = 7

6x - 1/2y = 5

Add both equations:

2x + 1/2 y + 6x - 1/2y = 7 + 5

8x = 12

x = 1 1/2

Solving for y:

2x + 1/2y = 7

We know x, so lets input

2(1.5) + 1/2y = 7

3 + .5y = 7

0.5y = 4

y = 4*2

y = 8

Check with the other equation

6x - .5y = 5

6(1.5) - 0.5y = 5

9 - 0.5y = 5

0.5y = 9 - 5

0.5y = 4

y = 4 * 2

y = 8

If my answer is incorrect, pls correct me!

If you like my answer and explanation, mark me as brainliest!

-Chetan K

CAN SOMEONE PLEASE HELP ME AND SOLVE THE PROBLEM STEP BY STEP IF IT WAS DONE CORRECTLY I WIL PUT U AS BRANLIEST THANK YOU!!!

Answers

Answer: (5,3)

Step-by-step explanation:

y=x^2-10x+28

Complete the square.

y=(x-5)^2+3

Use the vertex form, y=a(x-h)^2+k to find the values of a, h, and k.

a=1

h=5

k=3

Find the vertex (h, k).

(5, 3)

Express the following division in the form of a + bi: (1+3i)(2+i)

Answers

Answer:

[tex](1 + i\,3) \cdot (2 + i) = -1 + i\,7[/tex]

Step-by-step explanation:

We proceed to present the procedure by algebraic means:

1) [tex](1 + i\,3)\cdot (2 + i)[/tex] Given

2) [tex]2\cdot (1 + i\,3) + i\,(1+i\,3)[/tex] Distributive and Commutative Properties

3) [tex]2 + i\,6 + i + i^{2} \,3[/tex] Distributive, Commutative and Associative Properties

4) [tex]2 + i\,7 -3[/tex] Definition of imaginary number/Distributive, Commutative and Associative Properties

5) [tex]-1 + i\,7[/tex] Commutative and Associative Properties

A 50 foot ladder is set against the side of a house so that it reaches up 48 feet. If Jack
grabs the ladder at its base and pulls it 4 feet farther from the house, how far up the
side of the house will the ladder reach now? (The answer is not 44 ft.) Round to the
nearest tenth of a foot.

Answers

1.4 feet

let me know if you get it right hope i helped!

Answer:

14

Step-by-step explanation:

a2+(48)2=

 502\,\,50^{2}502

Use the Pythagorean Theorem.

a2+2304=a^{2}+2304=a2+2304=

 2500\,\,25002500

Simplify.

−2304-2304\phantom{=}−2304=

 −2304\,\,-2304−2304

Subtract 2304 from both sides.

a2=a^{2}=a2=

 196\,\,196196

a2=\sqrt{a^{2}}=a2

​=

 ±196\,\,\pm\sqrt{196}±196

Square root both sides.

a=a=a=

 14\,\,1414

Simplify. Ignore negative root, as length must be positive.

Which of the following is equivalent to 3 cm??
A.
B.
6 mm2
30 mm
90 mm-
300 mm
C.
D.​

Answers

Answer:

30mm

Step-by-step explanation:

please answer its due tomorrow ill give brainiest no links or scam

Answers

Answer:

4:    1 2/3

5.  5/21     b. 1/4    c. 11/18   d.  3/20

6.  11/12     b. 10/12 < 11/12

7. 4/45

Step-by-step explanation:

4.     1/3 times 5

6.    just add

7. 1/5 minus 1/9

Answer:

hey this is my alt acount im doing this so i can give brainiest

Step-by-step explanation:

-\frac{1}{10}-x^3+x^4+3x

10
1

−x
3
+x
4
+3x

Answers

Answer:

Its really simple

Step-by-step explanation:

first you take the 10 then turn it into joe then mama

Answer:

-1/10 - 1x^3 + x^4 + 3x

Step-by-step explanation:

negative one tenth minus 1x exponet 3 plus x exponent 4 plus 3x

Find value of x
A.)46
B.)80
C.)15
D.) 26

Answers

The two angles are supplementary angles, which when added together equal 180 degrees.

6x -10 + 6x +10 = 180

Combine like terms:

12x = 180

Divide both sides by 12:

x = 180/12

x = 15

The answer is C) 15

You have 90 degree triangle and the adjacent is 24, the opposite is 32, and the hypotenuse is 40. What is the value of tan W rounded to the nearest hundredth, if necessary.

Answers

Answer:

sorry need points Step-by-step explanation:

sorry need points sorry need points

9. A building 62 feet tall casts a shadow 21 meters long. At what angle is the sun shining on the building?
A. 19.80
B. 18.71
C. 70.20
D. 71.29

Answers

First, we imagine a right triangle. Second, we are given that the building is 62 feet tall. Since the height of building is vertical straight - that means the height is opposite side of triangle. It also casts a shadow 21 meters long. Shadow should be the adjacent side because it casts on the ground and not on the air.

If we imagine the right triangle + the problem in our head, we'd get the triangle that the opposite side is 62 feet tall and the adjacent side is 21 meters long.

Use the tangent ratio to find the angle of sun shining.

[tex] \large \boxed{ \tan \theta = \frac{opposite}{adjacent} }[/tex]

Substitute adjacent = 21 and opposite = 62 in the ratio.

[tex] \large{tan \theta = \frac{62}{21} }[/tex]

For this part, you might need a calculator or trigonometric table because some values cannot be exactly found.

From the equation, we should get 71.288 which we round the answer to nearest hundreth and the answer would be 71.29

Answer

The answer is 71.29

Answer:

71.29°

Step-by-step explanation:

By question its given that , a 62 feet tall tree cast a shadow of 21 m long. We need to find the angle at which the sun is shining on the building .

For this we gonna imagine a right angle triangle . Where , the length of the shadow will be the base of the triangle and the height of the building will be the perpendicular on the triangle .

Here since we have got perpendicular and base , we can use the ratio of tan . We know that tan is defined as perpendicular by base , that is ,

[tex]\implies tan \theta =\dfrac{perpendicular}{base}[/tex]

where theta is the angle between the Ray of the the sun and the the shadow of the building.

Plug in the respective values , we have ,

[tex]\implies tan \theta =\dfrac{62}{21} \\\\\implies\theta = tan^{-1}\bigg( \dfrac{62}{21}\bigg) \\\\\implies \underline{\underline{\theta = 71.29^o }}[/tex]

Hence the required answer is 71.29° .

Which negative angle is equivalent to 285 degrees?
A.-85 degrees
B.-95 degrees
C.-65 degrees
D. -75 degrees

Answers

By definition of coterminal angles, the negative angle that is equivalent to an angle of 285° is equal to - 75° degrees. (Correct choice: D)

How to find a negative coterminal angle?

In this problem we must determine a negative angle coterminal to a given angle, two angles are coterminal when both have the same direction. Given that a complete revolution is done each 360°, we can derive an expression for angles coterminal to a given one:

θ' = θ + i · 360°     (1)

Where i is the index of the coterminal angle.

If we know that θ = 285° and i = - 1, then the negative angle is:

θ' = 285° + (- 1) · 360°

θ' = - 75°

By definition of coterminal angles, the negative angle that is equivalent to an angle of 285° is equal to - 75° degrees. (Correct choice: D)

To learn more on coterminal angles: https://brainly.com/question/23093580

#SPJ1

Help me please I really need help

Answers

Answer:

[tex]17\sqrt{2}[/tex] ft

Step-by-step explanation:

take angle b as reference angle

using sin rule

sin 45 = opposite / hypotenuse

[tex]\frac{1}{\sqrt{2} }[/tex] = 17 / BC

BC =17 / [tex]\frac{1}{\sqrt{2} }[/tex]

BC = 17 * [tex]\sqrt{2}[/tex]

BC =  [tex]17\sqrt{2}[/tex] ft

Answer:

BC = 17[tex]\sqrt{2}[/tex] ft

Step-by-step explanation:

Using the sine ratio in the right triangle and the exact value

sin45° = [tex]\frac{1}{\sqrt{2} }[/tex] , then

sin45° = [tex]\frac{opposite}{hypotenuse}[/tex] = [tex]\frac{AC}{BC}[/tex] = [tex]\frac{17}{BC}[/tex] = [tex]\frac{1}{\sqrt{2} }[/tex] ( cross- multiply )

BC = 17[tex]\sqrt{2}[/tex] ft

Question 1 of 10
Classify the following triangle. Check all that apply.
60
60
60
O A. Isosceles
B. Equilateral
O C. Scalene
O D. Acute
E. Right

Answers

Answer:

A, B, & D

Step-by-step explanation:

A. isosceles - 2 sides equal (actually) all sides are equal

B. equilateral - all sides equal and all angles are equal too

D acute - all angles are less than 90 degrees

Which expressions are equivalent to the one below? Check all that apply.
log 2 - log 6
A. log(2) + log(1/6)
B. log 2
C.log(1/3)
D. log 3

Answers

Answer:

is there  a picture or something i cant understand

Step-by-step explanation:

Answer:

C. log(1/3)

Step-by-step explanation:

log a - log b = log(a/b)

Find the circumcenter of a triangle ABC with vertices at A(2, 2), B(0, -4), and C(7, -1)
20 POINTS PEOPLE

Answers

Answer:

just sum up the 3 sides.

quick side note: I lost my previews writing bc I waited too long too post it :(

but long story short: you get the length of, say, AB by using pythagoras theorem.

[tex] \sqrt{ {2}^{2} + {6}^{2} } [/tex]

do this with BC and CA too

the 2 is just the distance in x-direction and the 6 is just the distance in y-direction

and since a²+b²=c², we need to take the root to get the distance between 2 points

see screenshot for summing up the three distances. the part out of screen is written out above

the sum is 21.011 length units

hope it helps. have a nice day.

feel free to ask questions.

and maybe, if I'm feeling really nice and a lil bored, i leave greeting answers under yours to enable posters to give gives brainliests. if theres no 2and answer at least

The width of an extra large rectangle poster is 8 inches more than half its length.The area of this poster is 306 square inches

Answers

Answer:

a² + 16a - 612 = 0

Step-by-step explanation:

Here is the complete question :

The width of an extra large rectangle poster is 8 inches more than half its length. The area of this poster is 306 square inches

Write an equation in one variable that could be used to find the number of inches in the dimensions of this poster.

Area of a rectangle = length x width

Let length = a

width = 8 + 1/2a

Area =  (8 + 1/2a) x a

306 =  (8 + 1/2a) x a

multiply both sides of the equation by 2

612 = 16a + a²

a² + 16a - 612 = 0

the dimensions of the poster can be determined using quadratic formula

(ii) 3x - 5y - 4 = 0 and 9x = 2y +7​

Answers

Step-by-step explanation:

3×‐9×-5y-2y =0-7

-6x-7y =-7

Answer:

[tex]x = \frac{9}{13} \ , \ y = - \frac{5}{13}[/tex]

Step-by-step explanation:

                         3x - 5y = 4 ----------- ( 1 )

                         9x - 2y = 7 -----------( 2 )

                 ______________________

( 1 ) x 9  =>       27x - 45y = 36   ----------------- ( 3 )

( 2) x 3 =>        27x - 6y = 21     -------------------( 4 )

                  _____________________

( 3 )  =>          27x - 45y = 36

- ( 4 ) =>         -27x + 6y = - 21

                   ____________________

( 3 ) - ( 4 ) =>     0x - 39y = 15

                            -39y = 15

                                [tex]y = \frac{15}{-39} = -\frac{5}{13}[/tex]

Substitute y in ( 1 ) :

                     [tex]3x - 5y = 4\\\\3x - ( 5 \times -\frac{5}{13}) = 4\\\\3x + \frac{25}{13} = 4\\\\3x = 4 - \frac{25}{13}\\\\3x = \frac{52 - 25}{13}\\\\3x = \frac{27}{13}\\\\x = \frac{27}{13 \times 3}\\\\x= \frac{9}{13}[/tex]                        

80 25 x pls help me with this one

Answers

Answer:

25

Step-by-step explanation:

Answer:

x = 25

Step-by-step explanation:

Okay so as you can see there is an x shape and whenever you have that it means they are verticle angles. So we can conclude that 25 is verticle to x and so x is equal to 25.

Helpppp plz

What is the value of x?

Answers

Answer:

answer:

(x+34)+(2X+2)=180

3X+36=180

X=72

Step-by-step explanation:

Match the equation of circle with its center and radius.

Answers

Answer:

1- c=(-2,1) , R =9

2-c=(2,-1) , R=3

3-c=(-2,-1) , R=4

4-c=(2,1) , R=16

5-c=(0,0) , R=2

Other Questions
The middle school is 2.72 kilometers from Marsha's house and 1.54 kilometers from Ryan's house. How much farther does Marsha live from the middle school than Ryan? Group of answer choices What is the value of y in the equation 5x + 2y = 20, when x = 0.3? The chef at a restaurant bought 37 pounds of salad for $46.25. How much did she pay for 1 pound of salad? Show all work and explain your reasoning.Answer in paragraph You roll a six-sided number cube and flip a coin. What is the probability of rolling a number less than 2 and flipping heads? Write your answer as a fraction in simplest form. The probability is . A triangle has sides with lengths of 11 inches, 15 inches, and 18 inches. Is it a right triangle? PLEASE HELP ASAP!! THANK YOUEXPLANATION = BRAINLIEST & 5 STARS From the library, one car heads north, and another heads east. Some time later, the northbound car has traveled 17 miles, and the eastbound train has traveled 144 miles. The two cars, measured in a straight line, are ___ miles apart. Calculate the molality of a solution that contains 51.2 g of naphthalene, C10H8, in 500 mL of carbon tetrachloride. The density of CCl4 is 1.60 g/mL. If I have 3 classes and school ends at 2:18 P.M. How long should I work on each class for. The kidneys are organs of the body that work to maintain homeostasis. Which of the following is most likely to occur if a persons kidneys suddenly stop functioning?(A) a decrease in blood volume(B) an increase in urine production(C) a decrease in mass of the stomach(D) an increase in toxin levels in the blood How many significant figures are there in the number 10.8?A. 4B. 2C. 3D. 5 A concert loudspeaker suspended high off the ground emits 26.0 W of sound power. A small microphone with a 1.00 cm2 area is 53.0 m from the speaker. Part A What is the sound intensity at the position of the microphone Which statement accurately describes the role of ocean waters and seaice in Earth's system?Select one:O Both ocean waters and sea ice reflect the sun's energy.Both ocean waters and sea ice absorb the sun's energy.Ocean waters absorb the sun's energy. Sea ice reflects the sun's energy.Sea ice absorbs the sun's energy. Ocean waters reflect the sun's energy.Gc The original price of a bicycle was $400. Now it is on sale for $300. What percentage of the original price was the markdown? midpoint formula (7, 4) , (9, -1) how does a scientist answer a scientific question Can someone help me with this How was the decision in Dred Scott v. Sandford changed? Be sure to provide the name and a brief description of the congressional act, constitutional amendment, or subsequent Supreme Court ruling that overturned the decision The defensive coaches had hoped that the average number of tackles of the opponents QB would increase during the game in a distribution of 15% of the tackles in the first quarter,15% in the second,30% in the third, and 40% in the fourth. The mean number of tackles for the 2007 season was as follows: 37, 35, 73, 40. Did the mean sacks per game fit the desired distribution? Let a = 0.10. Given m||n, find the value of x.kt(6x+5)(7X-40 Given the following data: Selling price per unit $ 2.00 Variable production cost per unit S $ 0.30Fixed production cost $ 6,000Sales commission per unit $ 0.20Fixed selling expenses $ 1,500 How much is the break-even point in dollars? A) $6,000 B) $4,000 C) $8,000 D) $10,000 E) None of the above